5.5. Simetrias de processos de Wiener

Processos de Wiener possuem algumas simetrias importantes.

Invariância por reflexão

Considere um processo de Wiener padrão \(\{W_t\}_{t\geq 0}\). Então o processo \(\{V_t\}_{t \geq 0}\) definido por \(V_t = - W_t\) também é um processo de Wiener padrão. De fato, observe, inicialmente, que \(V_0 = -W_0 = 0\). Agora, se \(0 \leq t_0 \leq \cdots \leq t_k\) então os passos \(\{W_{t_j} - W_{t_{j-1}}\}_{j=1}^k\) são independentes. Como \(V_{t_j} - V_{t_{j-1}} = - (W_{t_j} - W_{t_{j-1}})\), então os passos \(\{V_{t_j} - V_{t_{j-1}}\}_{j=1}^k\) também são independentes. Além disso, \(V_{t + \tau} - V_t = - (W_{t+\tau} - W_t) \sim -\mathcal{N}(0, \tau) = \mathcal{N}(0, \tau).\) Finalmente, como os caminhos amostrais \(t \mapsto W_t\) são continuos, então \(t \mapsto V_t = -W_t\) também os são.

Observe que, por conta dessa simetria,

\[ \mathbb{P}(|W_t| \geq r) = 2\mathbb{P}(W_t \geq r), \]

para todo \(r \geq 0.\)

Invariância por rescalonamento

Considere um processo de Wiener padrão \(\{W_t\}_{t\geq 0}\). Dado \(a > 0\), defina o processo \(\{V_t^a\}_{t\geq 0}\) por

\[ V_t^a = \frac{1}{a}W_{a^2t}, \quad t \geq 0. \]

Então \(\{V_t^a\}_{t\geq 0}\) também é um processo de Wiener padrão. Caso \(\{W_t\}_{t\in [0, T]}\) só esteja definido em um intervalor \([0, T]\), \(T > 0\), então obtemos um processo de Wiener \(\{V_t^a\}_{t\in [0, a^2T]}\) em \([0, a^2T]\).

É imediato verificar que

(i) \(V_0^a = W_0 = 0\);

(ii) quase sempre os caminhos amostrais \(t \mapsto V_t^a(\omega) = (1/a)W_{a^2 t}(\omega)\) são contínuos; e

(iii) os incrementos \(V_{t_j}^a - V_{t_{j-1}}^a = (1/a)W_{a^2 t_j} - (1/a)W_{a^2 t_{j-1}}\), \(j = 1, \ldots, n\), \(0 \leq t_0 < t_1 < \ldots < t_n\), são independentes.

Quando à lei de distribuição dos incrementos, temos

(iv) Para todo \(\tau>0\), vale

\[ V_{t + \tau}^a - V_t^a = \frac{1}{a}\left(W_{a^2(t + \tau)} - W_{a^2t}\right) \sim \frac{1}{a}\mathcal{N}(0, a^2\tau) = \mathcal{N}(0, \tau). \]

Isso completa a demonstração dessa simetria.

Crescimento sublinear

Seja \(\{W_t\}_{t\geq 0}\) um processo de Wiener padrão. Então vale, quase certamente,

\[ \lim_{t \rightarrow \infty} \frac{W_t}{t} = 0. \]

Convergência em probabilidade

É fácil mostrar que a convergência ocorre em probabilidade. De fato, temos

\[ \mathbb{E}\left[ \frac{W_t}{t} \right] = \frac{\mathbb{E}\left[W_t\right]}{t} = 0, \]

para todo \(t > 0,\) e

\[ \mathrm{Var}\left(\frac{W_t}{t}\right) = \mathbb{E}\left[ \left(\frac{W_t}{t}\right)^2 \right] = \frac{\mathbb{E}\left[W_t^2\right]}{t^2} = \frac{t}{t^2} = \frac{1}{t} \rightarrow 0, \quad t\rightarrow \infty. \]

Portanto,

\[ \frac{W_t}{t} \rightarrow 0 \]

em probabilidade.

Convergência quase certamente

A demonstração de que a convergência ocorre quase certamente requer um pouco mais de trabalho. Observe, primeiro, que \(W_t/t \rightarrow 0\) quase certamente é equivalente a

\[ \mathbb{P}\left(\limsup_{t\rightarrow \infty} \left|\frac{W_t}{t}\right| \geq \varepsilon\right) = 0, \]

para \(\varepsilon > 0\) arbitrário. Isso, por sua vez, é equivalente a

\[ \mathbb{P}\left(\limsup_{n\rightarrow \infty} \max_{n-1 \leq t \leq n}\left|\frac{W_t}{t}\right| \geq \varepsilon\right) = 0, \]

onde \(n \rightarrow \infty\) ao longo dos números naturais. (Lembramos que, quase certamente, os caminhos amostrais são contínuous, de modo que podemos considerar o máximo acima, ao invés do supremo.) Pelo Lema de Borel-Cantelli, isso seque se mostrarmos que

\[ \sum_{n\in\mathbb{N}} \mathbb{P}\left(\max_{n-1 \leq t \leq n}\left|\frac{W_t}{t}\right| \geq \varepsilon\right) < \infty. \]

Para isso, precisamos de uma estimativa para cada termo da série. Observe que

\[ \begin{align*} \mathbb{P}\left(\max_{n-1 \leq t \leq n}\left|\frac{W_t}{t}\right| \geq \varepsilon\right) & = \mathbb{P}\left(\max_{n-1 \leq t \leq n}|W_t| \geq t\varepsilon\right) \\ & \leq \mathbb{P}\left(\max_{n-1 \leq t \leq n}|W_t| \geq n\varepsilon\right) \\ & \leq \mathbb{P}\left(\max_{0 \leq t \leq n}|W_t| \geq n\varepsilon\right). \end{align*} \]

Agora, um resultado essencial para a estimativa do termo acima é a estimativa maximal

\[ \mathbb{P}\left(\max_{0 \leq t \leq T}|W_t| \geq r \right) \leq \frac{2}{r^p}\mathbb{E}\left[ W_T^p \right], \]

para \(T > 0,\) \(r > 0\) e \(p\in \mathbb{N}\) arbitrários. Esta é uma versão contínua e estendida da desigualdade maximal de Kolmogorov, correspondente ao caso \(p=2.\) Essa desigualdade se aplica, mais geralmente, a qualquer Martingale e é conhecida como desigualdade maximal de Doob. O caso \(p = 4\) é suficiente para o nosso propósito. Mas antes vamos assumir que o resultado vale e concluir a demonstração de convergência. De fato, nesse caso, temos

\[ \mathbb{P}\left(\max_{n-1 \leq t \leq n}\left|\frac{W_t}{t}\right| \geq \varepsilon\right) \leq \frac{1}{n^4\varepsilon^4}\mathbb{E}\left[W_n^4\right]. \]

Como \(W_n/\sqrt{n} \sim \mathcal{N}(0, 1),\) podemos estimar

\[ \mathbb{P}\left(\max_{n-1 \leq t \leq n}\left|\frac{W_t}{t}\right| \geq \varepsilon\right) \leq \frac{2}{n^2\varepsilon^4}\mathbb{E}\left[\left(\frac{W_n}{\sqrt{n}}\right)^4\right] = \frac{6}{n^2\varepsilon^4}\mathbb{E}\left[\left(\frac{W_n}{\sqrt{n}}\right)^2\right] = \frac{6}{n^2\varepsilon^4}. \]

Desse modo,

\[ \sum_{n\in\mathbb{N}} \mathbb{P}\left(\max_{n-1 \leq t \leq n}\left|\frac{W_t}{t}\right| \geq \varepsilon\right) \leq \frac{6}{\varepsilon^4}\sum_{n\in\mathbb{N}} \frac{1}{n^2} = \frac{12}{\varepsilon^4} < \infty. \]

Portanto, como desejado, segue pelo Lema de Borel Cantelli que

\[ \mathbb{P}\left(\limsup_{t\rightarrow \infty} \left|\frac{W_t}{t}\right| \geq \varepsilon\right) = 0, \]

para \(\varepsilon > 0\) arbitrário, de modo que, quando \(t\rightarrow \infty,\), obtemos \(W_t/t \rightarrow 0\) quase certamente.

Desigualdade maximal

Para completar, vamos mostrar que

\[ \mathbb{P}\left(\max_{0 \leq t \leq T}|W_t| \geq r \right) \leq \frac{2}{r^4}\mathbb{E}\left[ W_T^4 \right], \]

para \(T > 0\) e \(r > 0\).

Pela simetria por reflexão \(W_t \mapsto -W_t,\) segue que

\[ \mathbb{P}\left(\max_{0 \leq t \leq T}|W_t| \geq r \right) = 2 \mathbb{P}\left(\max_{0 \leq t \leq T} W_t \geq r \right). \]

Pela continuidade dos caminhos amostrais, temos

\[ \mathbb{P}\left(\max_{0 \leq t \leq T} W_t \geq r \right) = \mathbb{P}\left(\sup_{t \in D} W_t \geq r \right), \]

onde \(D = \cup_{k\in\mathbb{N}}D_k\) é o conjunto de pontos da forma \(jT/2^k,\) \(j=0, \ldots, 2^k,\) \(k\in\mathbb{N},\) com

\[ D_k = \left\{ \frac{jT}{2^k}; \; j = 0, \ldots, 2^k\right\}, \]

ou seja, \(D\) é uma dilatação dos pontos diádicos. Esse conjunto \(D\) é denso em \([0, T]\) e a sequência \(\{D_k\}_{k\in\mathbb{N}}\) é monótona crescente. Assim,

\[ \mathbb{P}\left(\sup_{t \in D} W_t \geq r \right) = \mathbb{P}\left(\lim_{k\rightarrow \infty} \sup_{t \in D_k} W_t \geq r \right). \]

Logo, basta mostrarmos que

\[ \mathbb{P}\left(\sup_{t \in D_k} W_t \geq r \right) \leq \frac{1}{r^4}\mathbb{E}\left[ W_T^4 \right], \]

para todo \(k\in\mathbb{N}.\) Para isso, fazemos como na demonstração da desigualdade maximal de Kolmogorov. Dado \(D_k\), escrevemos \(t_j^k = jT/2^k,\) \(j=0, \ldots, 2^k,\) de modo que

\[ W_T = \sum_{j=1}^{2^k} (W_{t_j} - W_{t_{j-1}}). \]

Os passos \(X_j = W_{t_j} - W_{t_{j-1}}\) são independentes e com valor esperado nulo. Estamos, portanto, no contexto da desigualdade maximal de Kolmogorov, que nos dá

\[ \mathbb{P}\left(\sup_{t \in D_k} W_t \geq r\right) \leq \frac{1}{r^2}\mathbb{E}\left[W_t^2\right]. \]

No entanto, essa potência quadrática não é suficiente. Observamos, então, que \(X_j\) e \(-X_j\) têm a mesma distribuição \(\mathcal{N}(0, t_j - t_{j-1})\) e todos os \(X_j\) têm momentos finitos. Assim, vale

\[ \mathbb{P}\left(\sup_{t \in D_k} W_t \geq r\right) \leq \frac{1}{r^m}\mathbb{E}\left[W_T^m\right] \]

para qualquer potência \(m\in\mathbb{N}.\) Escolhendo \(m=4,\) obtemos

\[ \mathbb{P}\left(\sup_{t \in D} |W_t| \geq r \right) = 2\mathbb{P}\left(\lim_{k\rightarrow \infty} \sup_{t \in D_k} W_t \geq r \right) \leq \frac{2}{r^4}\mathbb{E}\left[W_T^4\right], \]

como desejado.

Invariância por inversão temporal

Considere um processo de Wiener padrão \(\{W_t\}_{t\geq 0}\) e defina o processo \(\{V_t\}_{t\geq 0}\) por

\[ V_t = \begin{cases} 0, & t = 0, \\ tW_{1/t}, & t > 0. \end{cases} \]

Então \(\{V_t\}_{t\geq 0}\) também é um processo de Wiener padrão em \([0, \infty)\). Vamos verificar isso.

Estado inicial

Por construção, temos \(V_0 = W_0 = 0\).

Continuidade quase sempre dos caminhos amostrais

Para quase todo \(\omega\), o caminho \(t \mapsto W_t(\omega)\) é contínuo. Daí tiramos que \(t \mapsto V_t(\omega) = t W_{1/t}(\omega)\) é contínuo em \((0, \infty)\). Além disso, segue do crescimento sublinear dos caminhos amostrais que, quase sempre,

\[ \lim_{t \rightarrow 0} V_t = \lim_{t \rightarrow 0} t W_{1/t} = \lim_{s \rightarrow \infty} \frac{W_s}{s} = 0. \]

Portanto, para quase todo \(\omega\), vale \(t W_{1/t}(\omega) \rightarrow 0\), quando \(t \rightarrow 0\), mostrando que \(t \mapsto V_t(\omega) = t W_{1/t}(\omega)\) é contínuo também em \(t = 0\), portanto é contíno em \([0, \infty)\), para quase todo \(\omega\). Isso mostra a continuidade quase sempre dos caminhos amostrais de \(\{V_t\}_{t\geq 0}\).

Independência dos incrementos

Quanto à independência dos incrementos, se \(0 < t_0 < t_1 < \ldots < t_n\), então \(1/t_n < \ldots < 1/t_1 < t/t_0\) e vemos que os incrementos \(V_{t_j} - V_{t_{j-1}} = t_jW_{1/t_j} - t_{j-1}W_{1/t_{j-1}}\), \(j = 1, \ldots, n\), também envolvem intervalos disjuntos, mas cada fator está multiplicado por um instante diferente, não sendo tão imediado deduzir a independência.

Vamos usar o fato de que esses incrementos formam uma normal multivariada e, com isso, a independência segue da independência dois a dois, que, por sua vez, segue se provarmos que as covariâncias são nulas.

Vamos, então, considerar quatro instantes distintos \(0 \leq t_1 < t_2 < t_3 < t_4\) e olhar para as covariâncias de dois incrementos disjuntos. Se \(t_1 > 0\), temos

\[ \mathrm{Cov}(V_{t_2} - V_{t_1}, V_{t_4} - V_{t_3}) = \mathrm{Cov}(t_2 W_{1/t_2} - t_1 W_{1/t_1}, t_4 W_{1/t_4} - t_3 W_{1/t_3}). \]

Distribuindo os termos e usando a propriedade \(\mathrm{Cov}(W_t, W_s) = \min\{t, s\}\) de processos de Wiener, obtemos

\[ \begin{align*} \mathrm{Cov}(V_{t_2} - V_{t_1}, V_{t_4} - V_{t_3}) & = t_2t_4\min\left\{\frac{1}{t_2}, \frac{1}{t_4}\right\} - t_2t_3\min\left\{\frac{1}{t_2}, \frac{1}{t_3}\right\} \\ & \qquad - t_1t_4\min\left\{\frac{1}{t_1}, \frac{1}{t_4}\right\} + t_1t_3\min\left\{\frac{1}{t_1}, \frac{1}{t_3}\right\} \\ & = t_2t_4\frac{1}{t_4} - t_2t_3\frac{1}{t_3} - t_1t_4\frac{1}{t_4} + t_1t_3\frac{1}{t_3} \\ & = t_2 - t_2 - t_1 + t_1 = 0. \end{align*} \]

Se \(t_1 = 0\), então

\[ \begin{align*} \mathrm{Cov}(V_{t_2} - V_{t_1}, V_{t_4} - V_{t_3}) & = \mathrm{Cov}(t_2 W_{1/t_2} - 0, t_4 W_{1/t_4} - t_3 W_{1/t_3}) \\ & = \mathrm{Cov}(t_2 W_{1/t_2}, t_4 W_{1/t_4}) - \mathrm{Cov}(t_2 W_{1/t_2}, t_3 W_{1/t_3}) \\ & = t_2t_4\mathrm{Cov}(W_{1/t_2}, W_{1/t_4}) - t_2t_3\mathrm{Cov}(t_2 W_{1/t_2}, W_{1/t_3}) \\ & = \frac{t_2t_4}{t_4} - \frac{t_2t_3}{t_3} \\ & = t_2 - t_2 = 0. \end{align*} \]

Como os incrementos são normais, isso mostra que quaisquer dois incrementos disjuntos de \(\{V_t\}_{t\geq 0}\) são independentes. Agora, considerando \(n\) incrementos consecutivos, o argumento acima mostra que eles são independentes dois a dois. E como são normais, isso implica neles serem mutuamente independentes.

Distribuição de probabilidades dos incrementos

Sejam, agora, \(t \geq 0\), \(\tau > 0\). Se \(t = 0\), então

\[ V_{t + \tau} - V_t = V_\tau = \tau W_{1/\tau} = \tau (W_{1/\tau} - W_0) \sim \tau \mathcal{N}(0, 1/\tau) = \mathcal{N}(0, \tau). \]

Se \(t > 0\), então

\[ \mathbb{E}[V_{t + \tau} - V_t] = 0 \]

e

\[ \begin{aligned*} \mathbb{E}\left[(V_{t+\tau} - V_t)^2\right] & = \mathbb{E}\left[\left((t + \tau)W_{1/(t+\tau)} - tW_{1/t}\right)^2\right] \\ & = \mathbb{E}\left[(t + \tau)^2W_{1/(t+\tau)}^2 -2(t+\tau)tW_{1/(t+\tau)}W_{1/t} + t^2W_{1/t}^2\right] \\ & = (t+\tau)^2\frac{1}{t + \tau} - 2(t+\tau)t\min\left\{\frac{1}{t + \tau}, \frac{1}{t}\right\} + t^2\frac{1}{t} \\ & = (t + \tau) - 2(t + \tau)t \frac{1}{t+\tau} + t \\ & = (t + \tau) - 2t + t \\ & = \tau. \end{aligned*} \]

Como o passo é uma normal, tendo média zero e variância \(\tau,\) temos

\[ V_{t + \tau} - V_t \sim \mathcal{N}(0, \tau). \]

De outra forma, podemos escrever

\[ V_{t + \tau} - V_t = (t + \tau)W_{1/(t + \tau)} - tW_{1/t}. \]

Somando e subtraindo \(t W_{1/(t + \tau)}\), obtemos

\[ V_{t + \tau} - V_t = (t + \tau)W_{1/(t + \tau)} - t W_{1/(t + \tau)} + t W_{1/(t + \tau)} - tW_{1/t} = \tau W_{1/(t + \tau)} + t (W_{1/(t + \tau)} - W_{1/t}). \]

Como \(W_0 = 0\), temos

\[ V_{t + \tau} - V_t = \tau (W_{1/(t + \tau)} - W_0) - t (W_{1/t} - W_{1/(t + \tau)}). \]

Vemos, com isso, que esse incremento é uma combinação linear das normais independentes

\[ W_{1/(t + \tau)} - W_0 \sim \mathcal{N}(0, 1/(t + \tau)), \quad W_{1/t} - W_{1/(t + \tau)} \sim \mathcal{N}(0, \tau/t(t + \tau)). \]

Portanto, esse incremento é uma normal com média zero e variância

\[ \tau^2\frac{1}{t + \tau} + t^2\frac{\tau}{t(t + \tau)} = \frac{\tau^2t + t^2\tau}{t(t + \tau)} = \tau. \]

Ou seja,

\[ V_{t + \tau} - V_t \sim \mathcal{N}(0, \tau), \quad \forall t \geq 0, \;\tau > 0. \]

Invariância por translações

Considere, novamente, um processo de Wiener padrão \(\{W_t\}_{t\geq 0}\). Seja \(s > 0\) e defina o processo \(\{V_t^s\}_{t\geq 0}\) por

\[ V_t^s = W_{s + t} - W_s, \qquad t \geq 0. \]

Claramente,

\[ V_0^s = W_s - W_s = 0. \]

Dados \(t_0 < t_1 < \cdots < t_n\), temos

\[ V_{t_j}^s - V_{t_{j-1}}^s = W_{s + t_j} - W_s - W_{s + t_{j-1}} + W_s = W_{s + t_j} - W_{s + t_{j-1}}, \]

sendo evidente que os incrementos de \(\{V_t^s\}_{t\geq 0}\) são independentes.

Agora, dado \(\tau > 0\), temos

\[ V_{t + \tau}^s = W_{s + t + \tau} - W_s - W_{s + t} + W_s = W_{s + t + \tau} - W_{s + t} \sim \mathcal{N}(0, \tau). \]

Finalmente, como, para quase todo \(\omega\), o caminho amostral \(t \mapsto W_t(\omega)\) é contínuo, temos, em particular, que

\[ t \mapsto V_t^s(\omega) = W_{s + t}(\omega) - W_s(\omega) \]

é também contínuo. Isso completa a demonstração de que \(\{V_t^s\}_{t\geq 0}\) é um processo de Wiener padrão.

Exercício

  1. Mostre que a combinação convexa \(W_t = (1-\theta)W_t^{(1)} + \theta W_t^{(2)},\) \(0\leq \theta \leq 1,\) entre dois processos de Wiener independentes \(\{W_t^{(1)}\}_{t\geq 0}\) e \(\{W_t^{(1)}\}_{t\geq 0}\) também é um processo de Wiener.

  2. Seja \(\{W_t\}_{t\geq 0}\) um processo de Wiener padrão. Mostre que o limite abaixo vale quase sempre, para qualquer \(p > 1/2\).

\[ \lim_{t \rightarrow \infty} \frac{W_t}{t^p} = 0. \]

Last modified: May 03, 2024. Built with Franklin.jl, using the Book Template.